LSAT and Law School Admissions Forum

Get expert LSAT preparation and law school admissions advice from PowerScore Test Preparation.

 reop6780
  • Posts: 265
  • Joined: Jul 27, 2013
|
#13280
I had hard time understanding what the stimuli was trying to say when I first read it.

Now I suppose the conclusion is that since the number of deaths rose with elective surgery after five-week during which the number of deaths declined, this surgery must have been the factor contributing the high mortality before the five-week period.

(Am I right? :ras: )

To weaken the conclusion, there should be alternative factor contributing to the high mortality or sth that disproves that surgery was consistently leading to the high mortality before and after five-week period.

Answer A was appealing since it gives an idea that it was not necessarily surgery that led to high mortality before five-week period as it deals with less risks.

However I chose answer E because I thought it provides an alternative explanation for the high mortality.
Doesn't it provide an idea that the surgery was not the sole reason for the high mortality?
 Adam Tyson
PowerScore Staff
  • PowerScore Staff
  • Posts: 5153
  • Joined: Apr 14, 2011
|
#13284
It looks to me like where you got a little off track here was in your analysis of the conclusion of the argument. While it is absolutely implied that the elective surgeries were the cause of the higher mortality rate, that is not what the author explicitly concluded. Rather, his conclusion is the last sentence of this stimulus - that the risks of elective surgery had been incurred UNNECESSARILY often (emphasis added) prior to the five week period. That's what we want to weaken - the claim that the risks associated with those elective surgeries weren't necessary.

Answer A attacks that idea, telling us that those elective surgeries would have become necessary later, and that the risks would have been higher with delay. That means that the surgeries, while elective at the time, still represented a better management of risk than the inevitable alternative.

Answer E talks about mortality issues that may accompany any surgery, elective or emergent, and also tells us only that those issues are possible and not that they actually happen. It doesn't do anything to undermine the claim that the earlier elective surgeries represented unnecessary risk.
 akanshachandra
  • Posts: 16
  • Joined: Jun 16, 2017
|
#36112
So, I also chose E, and I don't understand how we are to assume that A says that elective surgery would have been necessary.. because nothing in the statement makes us believe so?
 Luke Haqq
PowerScore Staff
  • PowerScore Staff
  • Posts: 747
  • Joined: Apr 26, 2012
|
#36170
HI akanshachandra,

(A) states, "The conditions for which elective surgery was performed would in the long run have been life-threatening, and surgery for them would have become riskier with time." The conclusion of the argument is "before the five-week period, the risks of elective surgery had been incurred unnecessarily often in the area."

If (A) were true, some of the people who received "elective" surgeries before the five-week period might have incurred mortality risks by opting for that surgery, but if they did not have the surgery, (A) is saying they would be taking on another set of mortality risks. The fact that such people would be making a trade-off in mortality risks calls into question the assertion that the risks from those elective surgeries "had been incurred unnecessarily often."

(E) doesn't reference the conditions/elective surgeries before the five-week period, yet that time period is what the conclusion is about. Since it doesn't reference that timeframe, (E) can't weaken the argument that elective surgeries during that period "had been incurred unnecessarily often."
 LSAT2018
  • Posts: 242
  • Joined: Jan 10, 2018
|
#44456
If I see this in terms of Cause/Effect, would it be that:
Unnecessary Risk (Cause) → Increasing Number of Deaths

I chose answer D because I thought the comparison showed that elective surgeries presented less of a risk. Then how is answer A a better choice?



(PS. The language in the stimulus and answers seem much harder than usual for this section as a whole!)
 Francis O'Rourke
PowerScore Staff
  • PowerScore Staff
  • Posts: 471
  • Joined: Mar 10, 2017
|
#44459
Hi LSAT2018,
Unnecessary Risk (Cause) → Increasing Number of Deaths
Can you tell me how you would express this relationship in your own words and where you saw it in the stimulus?

As for answer choice (D), I want to make sure that you first understood what Adam Tyson wrote above:
While it is absolutely implied that the elective surgeries were the cause of the higher mortality rate, that is not what the author explicitly concluded. Rather, his conclusion is the last sentence of this stimulus - that the risks of elective surgery had been incurred UNNECESSARILY often (emphasis added) prior to the five week period. That's what we want to weaken - the claim that the risks associated with those elective surgeries weren't necessary.
So in order to weaken this argument, we want an answer choice that will lead us to believe that those surgeries were in fact necessary. Once you understand this, answer choice (A) should look like a great candidate for the correct answer.

Answer choice (D) does not actually tell us that the speaker is mistaken and the the surgeries should have been performed. It seems to tell us that these surgeries are not so risky, but the answer choice qualifies this statement by telling us that they are less risky in general.

We don't know if this hospital was performing the elective surgeries that were exceptionally risky, or if they were performing the safest ones in the world. Both cases are possible. However since we know from the premises that these surgeries were causing a significant number of deaths, I can't say that these elective surgeries were safe and carried no risks of death.

I believe that that is what this answer choice is trying to get you to believe. After all, if these surgeries are safe then the risks were not incurred unnecessarily, because there are few risks. But this is ignoring the facts presented in the stimulus: people in this hospital are dying from elective surgeries. Who cares how safe elective surgeries are in general if we know how often people die while undergoing these procedures in the hospital we care about?
 T.B.Justin
  • Posts: 194
  • Joined: Jun 01, 2018
|
#47921
I was attracted to answer choice D because I looked at it initially as if elective surgery is less risky than emergency procedures, it could be true that emergency procedures were the cause of high mortality rather than elective surgeries.
 Adam Tyson
PowerScore Staff
  • PowerScore Staff
  • Posts: 5153
  • Joined: Apr 14, 2011
|
#47997
Careful there, T.B.Justin! Just because elective surgeries are "less risky" than emergency surgeries doesn't mean that they aren't risky! The argument is that unnecessary risks of elective surgeries were contributing to the mortality rate prior to the five week period. Not that those surgeries were the sole, or even the primary, cause of death among surgical patients, but that they were contributing something to the death rate. Them being less risky overall really does nothing to weaken that claim. If even one person died who would have lived had they not had the elective surgery, then that surgery may have been an unnecessary risk.

Answer A tells us that none of those elective surgeries were completely unnecessary, because they all would have become emergencies at some point down the road, increasing the risks further. That does the job! Conclusion undermined! Unnecessary risks weren't the culprit, because they would have become necessary in time.
 bigboyroeroe123
  • Posts: 10
  • Joined: May 04, 2020
|
#75444
Hi Powerscore,

I have a question regarding question stem. Here in this question, we are asked to find an answer choice weakening the conclusion. Most other weakening question stems usually ask us to weaken the argument. I wonder if there are substantial difference between these two types of question stems. Do we need to consider more about the logic relationship between the premise and the conclusion when asked to weaken the argument while just need to focus on the conclusion when asked to weaken the conclusion? Thanks!
 Luke Haqq
PowerScore Staff
  • PowerScore Staff
  • Posts: 747
  • Joined: Apr 26, 2012
|
#75937
Hi bigboyroeroe123,

Great question! On the one hand, to your question about whether there is a "substantial difference between these two types of question stems," I would say no. A question that asks you to weaken the conclusion will be asking the equivalent of one asking for you to weaken the argument. This is because an "argument" in these logical reasoning problems is the use of premises in order to arrive at a conclusion. Thus weakening the conclusion means the same thing as weakening the argument, which is used to reach that conclusion.

At the same time, on the other hand, you are right to wonder about the importance of considering "more about the logic relationship between the premise and the conclusion." It is certainly possible, for instance, that you'd encounter a stimulus with two different conclusions in it (a subsidiary and a main conclusion). In that case, the question stem could potentially ask about one or the other of the conclusions--I suppose in such an instance, it could be possible to weaken the subsidiary conclusion as a distinct matter from weakening the overall argument (and the main conclusion). I don't recall having seen such a question before that asks for an answer choice that weakens a subsidiary conclusion, but that's the main scenario in which I could see a difference between weakening an argument verses weakening a conclusion. Since you're unlikely to see such a weaken question with multiple conclusions, you should be safe assuming that "weaken the conclusion" versus "weaken the argument" is asking for the same thing.

Get the most out of your LSAT Prep Plus subscription.

Analyze and track your performance with our Testing and Analytics Package.